LSAT and Law School Admissions Forum

Get expert LSAT preparation and law school admissions advice from PowerScore Test Preparation.

User avatar
 Dave Killoran
PowerScore Staff
  • PowerScore Staff
  • Posts: 5853
  • Joined: Mar 25, 2011
|
#27114
Complete Question Explanation
(The complete setup for this game can be found here: lsat/viewtopic.php?t=49)

The correct answer choice is (D)

Because the second rule indicates that if the stand does not carry T then it must carry K (and by the contrapositive, if the stand does not carry K then it must carry T), the answer must be either K or T. For example, if the fruit stand attempts to carry only F, then by application of the second rule the stand does not carry T and therefore it must carry K. Thus, F cannot be the only kind of fruit that the stand carries. Since the answer must be K or T, and only T appears among the answers, answer choice (D) is correct. An alternate way of looking at the answers is to consider that O, P, and W are all sufficient conditions; that is, their occurrence automatically indicates that other kinds of fruit are carried by the stand. Thus, none of those fruits could be the only fruit carried by the stand.
 SherryZ
  • Posts: 124
  • Joined: Oct 06, 2013
|
#12766
Dec 2001 LSAT, Sec 4 Game 1, Q2:

I immediately excluded answer choices B, C and E because they are more restricted by rules. I hesitated between A and D. I felt they are both correct because based on rules, both F and T can be carried alone. :-?

Could you explain why A is wrong??

Thank you so much!!

---Sherry
User avatar
 Dave Killoran
PowerScore Staff
  • PowerScore Staff
  • Posts: 5853
  • Joined: Mar 25, 2011
|
#12771
Hi Sherry,

This question is all based on the second rule, which looks simple but is actually quite insidious. Let's look at this rule more closely.

The rule states that, "If the stand does not carry tangerines, then it carries kiwis." A lot of people glance at that and write it out as T :dblline: K. That is NOT the correct representation.

The rules as stated would be diagrammed as:


..... ..... ..... ..... T :arrow: K


The contrapositive is:


..... ..... ..... ..... K :arrow: T


Together, those two rules can be combined into:


..... ..... ..... ..... K :dblline: T


So, what does it all mean? Well, in direct terms, if T is not carried, then K must be carried, and via the contrapositive if K is not carried then T must be carried. Thus, if one of the two is not carried then the other must be carried. Therefore, you can never have a situation where both are not carried, which is what the super-symbolization at the end means. Also, it is possible for both to be carried under this rule.

Thus, in this game you always have to have at least one of either K or T carried in the fruit stand. In question #2 then, if only one kind of fruit is carried, it has to be K or T, and that means only (D) could be correct here. If that's still troubling you, imagine that is was only F that was carried. Well,t hat means T is not carried, which from the second rule means that K must be carried. See how that works? It's tricky!

You don't see a rule like this one all that often, but when you do, immediately pay attention to it because it will figure in the game in a significant manner (and will leave all sorts of destruction in its wake because more students never get fully comfortable with this one). and knowing how it works makes a question like this rather easy (but only if you are in the know about how the rules works; if you aren't this is a really hard question).

In your Logic Games Bible, take a look at pages 200-201 for a further discussion of this type of rule. For other readers, in the newest versions of the Logic Games Bible (June 2013 and later), take a look at pages 268-269. The concept appears elsewhere, but those two pages are a good start.

Please let me know if that helps. Thanks!
 pacer
  • Posts: 57
  • Joined: Oct 20, 2014
|
#17843
I am confused why answer choice A is incorrect

F only appears in the rule that

W :arrow: F or T

So F could appear by itself, and since it is a necessary condition, W may or may not occur

F is not linked to any of the other rules so the other variables can be included or not

based on this, I am thinking that F could be in the "in" group by itself

Can you go over this and discuss why A is the wrong

I was confused between answer choices A and D

Thanks
User avatar
 Dave Killoran
PowerScore Staff
  • PowerScore Staff
  • Posts: 5853
  • Joined: Mar 25, 2011
|
#17845
Hi Pacer,

What got you here was missing the implications of the second rule. Whenever you see a rule with a negative sufficient condition, be careful, because the absence of that variable triggers the rule.

In this case, if F was supposedly the only kind of fruit the stand carried, that would mean that K, O, P, T, and W would not be carried. That's mostly fine, but the second rules says that if the stand does not carry T, then it MUST carry K. So, you couldn't have just F.

That second rule means that at least one of T or K is always selected, thus the answer would have to be one of those two (and since K isn't a listed answer, it's T in this case).

Please let me know if that helps. Thanks!

Get the most out of your LSAT Prep Plus subscription.

Analyze and track your performance with our Testing and Analytics Package.